You are on page 1of 61
PHILIPPINE LAW SCHOOL ADMISSION TEST PhiLSAT ICE BOOKLET The CEM allows the reproduction of this practice booklet in any manner suitable to the PhiLSAT applicant. Other uses including selling or licensing copies, or posting to personal, institutional or third party website are prohibited. WY Educational ‘Measurement, Ine. G |ERAL DIRECTIONS The Philippine Law School Admission Test consists of four subtests. Each subtest contains multiple-choice items. For each item, select your answer from the options given. On your answer sheet, shade the circle marked with the letter of your chosen answer. For example, if your answer to an item is option C, then completely shade the circle marked C as shown below. Make sure your mark on the circle is dark. Avoid incorrect shading of circles as they may not be recognized as an answer. MARKING ANSWERS CORRECT MARK ®©O©@O®O INCORRECT MARKS =@® @©@O OO ®©OOO® ®O®KO® ®QOSO®O ®©OLOO ®OOO® Make sure you are marking the answer columns corresponding to the item number you are on. Mark only one answer for each item. If you want to change your answer, erase the first answer completely. Incomplete erasures will be interpreted as another answer thereby producing “multiple answers.” Items with multiple answers are automatically considered wrong Do not write anything on this test booklet. Use the blank pages of your answer sheet for your seratch work. Follow carefully the specific directions for each subtest or section. When you finish a subtest, proceed to the next until you have completed the entire test. DO NOT TURN THIS PAGE UNTIL YOU ARE TOLD TO DO SO. TEST A. COMMUNICATION AND LANGUAGE PROFICIENCY Section 1. Identifying Sentence Errors Directions: Read each sentence carefully but quickly, paying attention to the underlined word or Iphrase. Each sentence contains either a single error or no error at all. If the sentence contains an error, sch ct the underlined word or phrase that must be changed to make the sentence correct. Ifthe sentence is correct, select option D. {In choosing your answers, follow the requirements of standard written English 1 Lwas paying my bill in a restaurant when my childhood best friend suddenly come to A B have a short chitchat with me. No error c D Marco and Alea had been close friends for more than a decade, but people who knew them A thought that her relationship was something beyond friendship. No error B c D ‘The manager said that John needed to change his ways because he often came late, A failed to complete his tasks on time, and B ‘Most of my cousins wanted to be a teacher, except Santino who wanted to be an engineer. A B c No error D The supervisor and me would always discuss if we need to check the items so that A B we could avoid unexpected circumstances. No error c D We bel eve that it is you who has committed a grave mistake for which a A B Cc sincere apology should be extended. No error D BK PhiL SAP BA_0817, GO ON TO THE NEXT PAGES 10. LL 12 1B. 14, While the Middle Ages produced many great writers, Dante Alighieri the iconic author of A the Divine Comedy, is more celebrated than any writer from that period, No error B Cc D At the forum, the candidate said that he/she did not have nothing to offer but a promise to A B produce more employment opportunities in the country. No error Cc D ‘Matthew's potential to be an eloquent speaker was evident in his speech which won the A B admiration of not a few of his batchmates. No error c D A mother who knows the original value of an item can't help questioning the price of A B the same product when advertised on television, No error c D Some students today readily post their opinions and statuses on facebook, twitter, A or instagram: but others, for diverse reasons, choose to post using viber. No error B Cc D ‘The voters think Lovely would have won the election if she hasn't become haughty A B fo error D While attending the University, I used to have three roommates ~ one was an engineer, A B who wrote for the local dailies, and the third was a teacher, No error D ‘Two days before my father’s death, he complained that he could not hardly breathe, so A B we had to take him to the hospital. No error c D BK PhiL SAP BA_0817, GO ON TO THE NEXT PAGES 15. Neither Dan nor Jan thought they would attain all of those achievements in the company A B in spite of all the difficulties and challenges they encountered, No error Cc D 16. The police concluded that the suspect in the most recent incident of theft on campus was A B him based on the evidence gathered, No error c D 17. Kurt played his instrument so bad during the concert that the conductor decided to A pull him out in the middle of the program, No error B Cc D 18. The community officer, together with the other member, stroll around the village A B to identify the needs and problems of the villagers, No error Cc D 19. Lopez's hotel has more guests than Chua does because of its location and excellent A B c 20. fone eats prudently and is mindful of the amount of calorie intake, he or she could retain A B their desirable weight. No error Cc D 21. The Senate and the House of Representatives were arguing of whether she was qualified A B for the Witness Protection Program, No error c D 22. Asa people, we are known for our bravery and patience, having put up on with almost A B c four centuries of colonial domination. No error D BK PhiL SAP BA_0817, GO ON TO THE NEXT PAGES Section 2. Sentence Completion Directions: Choose the word or phrase that, when inserted in the sentence, best fits the meaning of the sentence as a whole. 23. 24, 25. 26. 21. Cecilia's mother from Switzerland 30 years ago, and she found a haven n the Philippines. (A) emigrated (B) immigrated (C) has emigrated (D) has immigrated ‘After seeing the movie, Andrea took her eyeglasses off and put them her lap. (A) to (B) on © in (D) at Contemporary Manila, with its images of urbanization and poverty, is from Old Manila, once romantically described as the Queen City of the Pacific (A) afarery (B) grain of salt (C) the last straw (D) the wrong tree the presenter had rehearsed the part she thought the most difficult, the participants did not appreciate her effort and went home unhappy. (A) Since (B) Because (©) Ionly (D) Even though Yosef presented to the team than what the company purchased three years ago, (A) a powerfuller device (B) the powerfuller devi (C) amore powerful device (D) the more powerful device BK PhiL SAP BA_0817, GO ON TO THE NEXT PAGES 28. 29. She was answering her assignment on historical background of a short story she discovered she was in the wrong page. (A) after (B) but (©) and (D) when ‘After a tight and exhausting schedule yesterday, Ramon in bed since early this morning. (A) lay (B) lying (C) has lain (D) had lied The passengers are informed that they have the next four hours leisure, and can go wherever they wish. (A) at (B) by (©) on (D) as Because the problem is rather insoluble, even those who initially wanted to take it up have now dropped it like a (A) _ penny for your thoughts (B) piece of cake (C) spilt milk (D) hot potato We are expected to our outputs on or before Thursday next week. (A) tuto (B) tum off (©) tum in (D) tum into She was (the) among the researchers in this institution, despite her formidable credentials. (A) humbler (B) humblest (C) more humble (D) most humble BK PhiL SAP BA_0817, GO ON TO THE NEXT PAGES 34. She was the village when she found out that there was a bakeshop and craft shop near the clubhouse. (A) walking through (B) walking around (C) looking beside (D) looking along 35. Assuring the people that the truth shall always prevail, each member of the panel of judges swears that he or she shall be (A) _ interested (B) uninterested (C) disinterested (D) misinterested 36. Because the documents presented are substantial, and they will contribute to better the system of the organization, the Board agrees your proposal. (A) to @®) in (©) on (D) at 37. Andrea has eamed the highest academic average in the history of the university; , She cannot attend the commencement exercises next week because of serious family concerns. (A) however (B) although (©) moreover (D) furthermore 38. He is than any other student of history that I know. (A) _ eruditer (B)_ eruditest (C) more erudite (D) most erudite 39, Most students in some foreign countries are the Study Now, Pay Later Program of the government to earn their college degrees. (A) _ banking off (B) banking on (C) banking to (D) banking in BK PhiL SAP BA_0817, GO ON TO THE NEXT PAGES Directions: portion is correct or needs revision. If the sentence needs no revision, choose option D. Section 3. Improving Sentences Read each sentence carefully but quickly and determine whether the underlined In choosing your answers, follow the requirements of standard written English. Pay attention to grammar, choice of words, sentence construction, and punctuation, Choose among the options which revision results in a sentence that is clear and precise. 40. My colleague was upset all day he did not get the promotion he was hoping for. @) (B) ©) ©) My colleague was upset all day, he did not get the promotion My colleague was upset all day: he did not get the promotion My colleague was upset all day because he did not get the promotion No change 41. Since Judy was absent because of severe migraine. I took notes so that she would not miss anything from today’s lecture. “” @) © @) 42. Being defective, the guest was advised by I took notes so that Judy would not miss anything from today's lecture; since she ‘was absent. T took notes so that Judy would not miss anything from today's lecture, Since she was absent because of severe migraine. Since Judy was absent because of severe migraine, I took notes so that she would not miss anything from today's lecture. ‘No change the front desk staff not to use the computer stationed at the lobby area of the hotel, (A) The front desk staff, being defective, advised the guest not to use the computer stationed at the lobby area of the hotel. (B) The front desk staff advised the guest not to use the computer stationed at the lobby area of the hotel because it was defective. (C) Being defective, the computer was not used by the guest stationed at the lobby area, as advised by the front desk staff. (D) No change 43, Because half of the class has been absent today, we shall have the examination on Friday instead. (A) _ Because half of the class are absent today (B) Because half of the class were absent today (C)__ Because half of the class is absent today (D) No change BK PhiL SAP BA_0817, GO ON TO THE NEXT PAGES 10 44, Everybody feels that, contrary to rumors, the new policy will prove beneficial to the organization, it will result in an increase in sales and greater sense of satisfaction on the part of the workers. (A) and will result (B) but will result (C)_ yetwill result (D) No change 45. If the circumstances will become favorable to the purchase of new equipment, then we shall immediately make the recommendation, (A) _ become, shall (B) becomes, shall (C)_ will become, will (D) No change 46. Located in the Summer Capital of the Philippines, the new museum showcases notable works of both seasoned artists and novices. (A) The new museum showcases the notable works of both seasoned artists and novices located in the Summer Capital of the Philippines. (B) Located in the Summer Capital of the Philippines, the notable works of seasoned artists and novices are showcased in the new museum, (C)__ The notable works of seasoned artists and novices located in the Summer Capital of the Philippines are showcased in the new museum, (D) No change 47. Some historical accounts claim that the Philippines was more devastated than any Asian country after the Second World War. (A) _ devastated more than any Asian country (B) more devastated than any other Asian country (C) most devastated than any other Asian country (D) No change BK PhiL SAP BA_0817, GO ON TO THE NEXT PAGES u 48. After reaching a weight of 78 kilograms, the doctor advised the female patient to avoid meat and to drink 2 — 3 liters of water every day. (A) After reaching a weight of 78 kilograms, the female patient was advised by the doctor to avoid meat and to drink 2 — 3 liters of water every day. (B) After reaching a weight of 78 kilograms, the doctor was advised by the female patient to avoid meat and to drink 2 —3 liters of water every day. (©) The female doctor, after reaching a weight of 78 kilograms, advised the patient to avoid meat and to drink 2 —3 liters of water every day. (D) No change 49. ‘The farmer sadly bid farewell to his son, who desires to find work in another land and reciprocate his father’s sacrifices, a fresh graduate, (A) _ The farmer sadly bid farewell to his son, a new graduate who desires to find work in another land and reciprocate his father's sacrifices. (B) A fresh graduate who desires to find work in another land and reciprocate his father’s sacrifices, the farmer sadly bid farewell to his son. (C) The farmer sadly bid farewell to his son, who desires to find work in another land, a fresh graduate, and reciprocate his father’s sacrifices. (D) No change 50. The employers had no evident and clear solution to the problems of their employees there i jion and be by -d to bring up the said concerns, (A) The employers had no evident and clear solution to the problems of their employees, there was no increase in compensation and benefits because no one initiated to bring up the said concerns. (B) The employers had no evident and clear solution to the problems of their employees: there was no increase in compensation and benefits because no one initiated to bring up the said concerns. (C)__ The employers had no evident and clear solution to the problems of their employees; there was no increase in compensation and benefits because no one initiated to bring up the said concerns. (D) No change BK PhiL SAP BA_0817, GO ON TO THE NEXT PAGES 12 TEST B. CRITICAL THINKING Section 1. Logical Reasoning Directions: In this section, brief conversations or short passages are given followed by one or two questions. These questions ask for the analysis and evaluation of the reasoning in the conversation or passage. It is conceivable that all of the four choices given for each question may be correct answers. The task is to select the best answer for each question, The best answer is the one which does not require assumptions that are irrelevant or inconsistent with the statements, ‘The proliferation of domestic conflicts and issues, particularly those related to the abuses a wife experiences from her husband, has always been associated with the traditional gender roles men and women assume in the home, Men are considered as the heads of the family and providers. of material goods, while women, on the other hand, are reduced to being their husbands' helpmates in terms of performing household duties. 1. Which of the following conclusions would the author of the extract above most likely agree with? (A) Order in the family necessitates leadership of the patriarch. (B) Modern society still advocates traditional gender roles. (C) Women are tied down to houschold duties. (D) Gender is socially constructed, Recent strides in education research have shown the need for a more criterion-based rather than norm-based grading system. Traditionally, norm-based grading is used to show student ranking, thus instilling a sense of competition among students. While this may be the case, norm-based grading does not clearly show the specific strengths or areas of improvement of individual students. Furthermore, such a system does not seem to be adequate anymore in the information age as our current social context necessitates cooperation rather than competition. Hence, criterion-based grading seems to be the answer to this dilemma as this does not rank students, which to some, seem to be a measure of a person's innate worth, Moreover, in this age of specializations, it can specifically point out students’ strengths and challenges, thereby allowing, educators and employers alike a clearer view of one's capabilities. 2. Which of the following is the best way to improve one’s argument against the limitations of norm-based grading? (A) Cite specific instances in which norm-based grading was used. (B) Provide evidence of its mismatch with the current social context. (C)_ Explain the use of eriterion-based grading in finding course specializations, (D) Describe how norm-based grading can potentially devalue students’ competitive spirits BKL PhILSAT BA 0817 GO ON TOTHER NEXT PAGE 3. Based on the passage, which of the following claims is considered erroneous? (A) Norm-based grading develops competitive attitudes. (B)_ The information age demands collaboration. (C)_ Employers tend to look at specific abilities. (D) A student's rank depicts his innate worth The United Nations (UN) Secretary General has condemned any form of state punishment that either takes the life of the convict or causes severe physical or psychological trauma, He argues that though the convicts’ actions may have caused harm in the past, nothing can change the fact that they are still human beings capable of doing good. Furthermore, they are also entitled to basic human rights regardless of what they have done. He even goes further to point out that several studies have shown that harsher punishment, historically, does not automatically result in fewer crimes 4, What is the most likely objective of someone who quotes the UN Secretary General as he begins to spell out his arguments? (A). To present factual observations. (B) To use an authoritative figure to substantiate his points. (C)_ To highlight the universality of the argument proposed, (D) To appeal to emotions to convince his audience to favor him, Textbooks are already a thing of the past, and therefore should be phased out. They were designed to cater to the need of the industrial age for knowledge to be conveniently pre-selected and packaged to prepare the population for clearly delineated roles. Nonetheless, in an information age such as outs, it seems that textbooks have already lost their significance. The Intemet’s ubiquity has now allowed virtually everyone to access and even contribute information to the World Wide Web. Furthermore, the digital age has also challenged traditional careers, giving birth to new job functions at an unprecedented rate, Hence, to prepare our students more effectively for the future, we should stop using textbooks and start teaching our kids to access information in is for the author's argument in phasing out textbooks? (A). The renewed social context. (B)_ The accessibility of textbooks. (C)_ The technology gap among students. (D) The difficulty of finding information in textbooks. 6. Which of the following measures would the author most likely agree with? (A)_ Require students to have digital devices. (B) Give assignments wherein answers can be found on the Internet. (C)_ Maximize the use of educational technology in classroom teaching. (D)_ Seta daily limit for the use of the Internet, say for example, four hours a day. ‘BKL PhiLSAT, BA 0817 GO ON TO/THE,NEXT PAGED 14 While poetry and prose are often classified under the same art form, literature, it would be a mistake to label them as one and the same. In prose, meaning is exhibited by following the formal structures of language, like grammar. Poetry, on the other hand, breaks these very structures that normally encapsulate linguistic meaning so that it may give birth to a limitless aesthetic experience. It is therefore safe to say that poetry has a lot more in common with abstract painting than with prose, 7. Itcan be inferred that poetry and prose are different, (A). by virtue of their length (B)_ by virtue of their corresponding authors’ writing styles (C)_ in terms of how ideas are expressed (D) in terms of descriptive language used 8, Which of the following best interprets the last statement? (A) Poetry is as boundless as abstract painting, (B) Poetry and abstract painting do not exhibit meaning, (C)_ Poetry should be classified under visual arts and not literature, (D) Poetry is comparable to abstract painting because they both employ obscurity and ambiguity he prevalence of information technology in our era brought with it both positive and negative consequences. One of the negative ones is the advent of cybercrimes. Criminality in cyberspace is much more difficult to detect because of the perpetrators’ easy access to anonymity. Herein, I propose that the most efficient solution to this is for the authorities to confidentially monitor the user activity of every citizen so that criminality becomes easily detectable. In this respect, users are guaranteed safety and security in the cyberspace. 9. Which of the following illustrates the argument's tendency for contradiction? (A). The commission of cybercrime results from anonymity, one of the features that make cyberspace attractive. (B) The passage indicates that there is online anonymity, whereas the proposed solution refutes this by suggesting that authorities should monitor the user activity. (C)_ The concluding statement suggests that there is guaranteed safety and security in allowing authorities to invade privacy of users. (D) The proliferation of information was generated out of good intentions, but ended up giving rise to new sets of problems. BKL PhILSAT BA 0817 GO ON TOTHER NEXT PAGE 15 ‘A day before the presidential election, Michael has difficulty deciding on whether or not he would support the United Party or go for the Nationalist candidate. Little did he know, the Nationalist Party equipped a tampering device in his brain to monitor his cognitive activity If he decides to vote for the United candidate, the device will alter his choice. If, however, he decides to go for the Nationalist candidate, the device will allow him to. Michael voted for the Nationalist without any influence from the device. 10, Supposing that Michael found out that his brain was tampered with a device, and still decided to vote for the Nationalist candidate, this conveys that freewill is (A). doing the right decision (B) having the option to do otherw (C)_ relative to the number of possible choices (D) possible even when one is deprived of an alternative choice If there was ever a single best solution to traffic, that would be trains. Since they are ran and monitored from information bases, their coming and going are synchronized unlike vehicles on the road, which may come and go as they please, thus causing traffic jams. Moreover, since trains have a bigger camying capacity, they can accommodate more commuters. Furthermore, freight trains can transport all sorts of construction materials and heavy equipment, freeing up our roads from trucks. Undoubtedly, trains are the answer we have been waiting for, all highly industrialized nations are equipped with sophisticated railways. So should we. 11. Which of the following may be considered as a weakness on the argument to commission more trains? (A) It argues that using freighter trains would lessen the number of cargo trucks. (B) It assumes that only trains can be synchronized and not vehicles on the road. (C)_Itonly speaks about commuters and freighters, and not owners of private vehicles. (D) It presupposes that trains have a bigger carrying capacity than vehicles on the road. 12. The last two sentences mention that the highly industrialized nations’ sophisticated railways justifies the installation of our railway network. Which of the following assumptions behind this claim is potentially flawe (A). Trains are very instrumental in paving the way for economic progress. (B) Following everything in highly industrialized nations do guarantees our own progress. (©). Trains are mechanized and automated vehicles that can be controlled with more precision, (D) Our growing population, which is at par with other nations, deserves to enjoy more efficient transportation systems. ‘BKL PhiLSAT, BA 0817 GO ON TO/THE,NEXT PAGED 16 Inher revolutionary work, The Second Sex, the French existentialist, Simone De Beauvoir, wrote the following statement: "One is not born but rather becomes a woman.” Beauvoir, in an effort to diminish the patriarchal forces that dominated the 20" century, was arguing for a reformation of ideology not just on the part of the dominant sex, the male, but also on the part of the so-called inferior sex, the female. Accordingly, the woman must strive to be her authentic self rather than settle for the values that the patriarchal society attributes to her. 13, Which of the following statements best explains the main idea of the passage? (A) Women inferiority was shaped by the existing societal values. (B) Feminism is the result of women being dominated by patriarchal forces. (C)_ Patriarchy was most rampant in the 20 century as it involved the participation of both sexes. (D)_ Inasmuch as men constitute patriarchy, women also have a substantial role to play in obtaining their authenticity. 14, What does the statement, "One is not born but rather becomes a woman," mean in this context? (A) A person goes through a gender identity crisis. (B) A woman is treated as a woman because of her physical sex. (C) Ittakes effort to become a woman, thus womanhood is not pre-given. (D) Womanhood is determined by both the biological and environmental conditions. Asa college professor for more than 50 years now, Ihave taught quite a number of students coming from diverse backgrounds, Incidentally, about 90% of all my Bachelor of Science students can exhibit above average performances in any sort of academic task I give them, whereas only 48% of all my Bachelor of Arts students can gamer the same performance for the same tasks. This is no doubt due to the high level of intelligence that the sciences require as opposed to that of the arts 15. Itis evident that the above argument is (A) strong, because the statistical data he presented clearly legitimizes his main point (B) strong, because his lifelong teaching experience grants him the authority to make a legitimate analysis of his gathered data (C)_ weak, because it heavily relies on statistics rather than on facts (D) weak, because the statistics does not factor in other variables, rendering the conclusion unsupported 16. The speaker would most likely agree that (A)_ science majors generally perform better in academics than art majors (B) about 40% of science majors are more intelligent than art majors (C) art majors are almost half as intelligent as science major (D) science courses are more difficult than art courses BKL PhILSAT BA 0817 GO ON TOTHER NEXT PAGE 7 ‘A wise man once told me that the best lawyers are those who can see beyond the borders of legal practice. The law, he says, should only serve as an instrument for realizing a higher, transcendental end, that is, the pursuit of justice. The law is not to be held supreme above all else but should be perceived as a means to concretize our moral ideals. 17. Itcan be inferred from the passage that a good lawyer should (A) have a high moral compass that shall be put into concrete action through the law (B) focus in knowing the law so long as justice is at the heart of his/her practice (C)_ have the will to violate the conventional borders that legal practice offers (D) be able to distinguish between what is moral and what is legal Consequentialism is an ethical theory that follows the dictum: "the end justifies the means." It rests on the assumption that, in order to achieve the good, one needs not look at the ways in which the good has been achieved, but should only focus on whether or not the desired end, that which corresponds to the greater good, is met. 18. Which of the following scenarios can be best justified by consequentialism? (A) A beautiful young woman married a rich old man for money. (B) Aman employed a child begging for work to feed his family, (C) Aman murdered his wife after learning that she had cheated on him. (D) Asingle mother left her son with her parents to have a better life abroad. The country's system of education has evolved to become very sympathetic of children. Corporal punishment is now perceived to be evil and thus renounced by the government. Back in my day, hitting was accepted as a disciplinary method so that students may be able to lean the hard way. Look at me now; I have relished a lifetime of success because of my acquired ability to endure pain and my resilience against suffering 19. It can be inferred that the speaker's argument is (A) strong, because it exposes the government's unreliability in producing resilient students (B) strong, because he presented his first hand life experience as a validating proof of the efficiency of the previous system of education (©) weak, because violence can never be justified under any circumstances (D) weak, because his life experience, which supposedly supports the main premise, does not represent that of everyone else's ‘BKL PhiLSAT, BA 0817 GO ON TO/THE,NEXT PAGED 18 While many have argued that the question of morality goes hand in hand with religiosity, evidence of it being otherwise is dominant in our history. The battle for the Promised Land and. the rise of the Crusades, the unending warfare between religious groups in the Middle East — all support the fact that when one renders dogmatism and religiosity as the highest virtue, and human dignity as only second, the violation of human rights becomes one’s imperative cause. In truth, one’s moral judgment is independent of one's religion. 20. What can be inferred from the passage? (A) So long as the cause is holy, it is justifiable to commit heinous acts (B) Morality and religiosity are not causally linked. (C)_ Morality and religiosity are two opposite poles. (D) Religion is the main driving force of heinous acts. 21. Which of the following demonstrates a FALSE inference about the passage? (A) Religiosity does not necessarily remove the possibility of committing immoralities. (B) Several wars in history are caused by conflicting religious principles. (C)_ The religious ones are more often than not the violators of morality. (D)_ One can have good moral judgment regardless of one's religion. I conducted an experimental research on the effectiveness of collaborative learning vis-a-vis direct instruction in teaching grammar. In my study, I looked at one control group and one variable group. The control group was taught using direct instruction while the variable group ‘was taught using collaborative learning. I then administered a multiple choice test to determine which group learned grammar better. Results show that the variable group got significantly higher scores. Since experiments yield definitive results as in the natural sciences, we can therefore conclude that the collaborative method is superior to direct instruction when teaching grammar. 22. Which of the following statements is valid based on the passage above? (A). Experiments yield definitive results. (B)_Anexperiment should involve variables. (C)_ Multiple choice tests best measure grammar learning. (D) Collaborative learning is better than direct instruction when teaching grammar. 23. Based on the evidence provided, the statement "collaborative method is superior to direct instruction when teaching grammat" is (A) conclusive since the methodology is an experiment and thus, results showing the superiority of collaborative method is expected to be precise (B) conclusive because the difference in scores between the control and variable groups prove the superiority of collaborative learning (C)_ not conclusive because the steps in the scientific method were not clearly followed in the experiment to prove the superiority of collaborative method (D) not conclusive because the evidence is insufficient to generate the conclusion that in all cases, collaborative learning is better BKL PhILSAT BA 0817 GO ON TOTHER NEXT PAGE 19 The notion of mental age is a long-standing myth in our culture. It excuses the labelling of children who are diagnosed with certain intellectual disabilities as somewhat 'mentally behind’ when compared to that of other children with the same chronological age. With the arrival of inclusive education, child rights advocates are now campaigning for the notion to be abandoned in schools. It is now the educator's task to employ the proper set of learning skills onto these children in order for them to cope with other students who are blessed to have a normal condition. 24, Which of the following best justifies the abandonment of the mental age notion? (A)_Itis a false and an unnecessary hindrance to the child's learning development. (B) Educators should do their duty rather than blame the children's condition. (C)_ Children with intellectual disabilities are likely to be bullied because of the label (D) Children with intellectual disabilities are offended by the label. Many of us still think of science as an irrefutable source of truth. We draw our understanding of reality based on its discoveries about the mechanisms that govern the universe. However, there is an unspoken danger that lies in presuming the irrefutability of science, as demonstrated by the philosopher Karl Popper. Accordingly, the definitive attribute of scientific theories lies in their refutability. Ifa theory becomes impossible to refute, it is most likely that the theory is pseudoscientific. 25. Which of the following best explains the main thesis of the passage? (A) Theories that seek to allude towards a single theory to unite all other sciences are pseudoscientific. (B) Scientists are not infallible — they are also capable of making mistakes. (C) Scientific theories leave room for the possibility of falsification. (D) Theories that can be falsified are scientific theories. ‘BKL PhiLSAT, BA 0817 GO ON TO/THE,NEXT PAGED 20 Section 2. Analytical Reasoning Directions: In this section, some sets of conditions or propositions are given. Each set is followed by questions based on the conditions or propositions. Drawing a diagram to visually establish linkages or relationships stated in the conditions may help in answering some of the questions. Select the best answer to each question from the choices given. A restaurant observes the following menu schedule: I. Burger is served on Mondays, Thursdays, and Fridays, all in the afternoon. IL. Pizza is served all day on Tuesdays and Fridays, IIL. Pasta is served on the mornings of Wednesday, and Thursday. IV. Chicken is served all day on Tuesdays and Wednesdays, V. Salad is served the whole day on Fridays. 26. At what day and time of the week is it possible to order the most number of dishes? (A). Friday afternoon (B) Tuesday afternoon (C) Thursday moming (D) Wednesday morning 27. Excluding weekends, at what day and time of the week is the restaurant presumably closed? (A) Wednesday afternoon (B) Tuesday afternoon (C) Thursday morning (D) Monday moming 28, Which of the following dishes may be ordered together? (A) Pizza and pasta (B) Pizza and salad (©) Burger and pasta (D) Chicken and burger 29. Allof the following dishes are offered solo at certain times of the week EXCEPT (A) chicken (B)_ burger (©) pasta (D) salad BKL PhILSAT BA 0817 GO ON TOTHER NEXT PAGE 21 ‘There is a wall clock at Lennon's room, which serves as the focal point of the wall. When the clock strikes 12, the hour hand points to a Van Gogh painting; at 9 o'clock, it points to a hanging bookshelf; at 3 o'clock, it points to his graduation picture; and at 6 o'clock, it points to a TV set. ‘There is a baby picture directly east of the Van Gogh painting. There is a lamp north of the bookshelf. Directly east of the baby picture is a Beatles poster. 30. Which of the following conditions is true when the clock strikes at 1:30? (A). The minute hand points to the lamp. (B)_ The minute hand points to the graduation picture. (C)_ The hour hand points to the baby picture. (D) The hour hand points to the Beatles poster. 31. Which of the following set of objects consist the north of the clock? (A) Van Gogh painting, book shelf, lamp, Beatles poster (B) Van Gogh painting, baby picture, lamp, Beatles poster (C) Beatles poster, lamp, graduation picture, TV set (D) Beatles poster, Van Gogh painting, baby picture, graduation picture 32. Which of the following sets consist of at least one object west of the clock, at least two objects east of the clock, and at least three objects north of the clock? (A)_ Beatles poster, Van Gogh painting, baby picture, lamp (B) Van Gogh painting, Beatles poster, lamp, TV set (C)_ graduation picture, book shelf, lamp, Beatles poster (D) lamp, Van Gogh painting, baby picture, graduation picture In a class of 40, 28 students are joining the local field trip while 19 students are joining the international field trip. In the same class, only five students expressed that they are not joining either of the trips. Those who are not joining the trip as well as those joining only the local trip are required to write a reflection paper. Those who are joining the international trip are required to write a research paper. 33. How many students are required to write only a research paper? (A) 5 (By) 7 (©) 16 (D) 35 34, How many students will write only a reflection paper? 19 (B) 21 © 28 (D) 35 ‘BKL PhiLSAT, BA 0817 GO ON TO/THE,NEXT PAGED 22 35. How many students will write both a reflection paper and a research paper? (a) 2 (B) 19 (© 23 (D) 28 Julia is making a schedule for her grocery store. She needs at least one person each for morning shift, midday shift, and night shift. Each employee is allowed only one shift per day. Taking into consideration the staff's shift preference and working relationships, she notes the profiles of the following personnel: ‘Anna and Edgar can work in the morning, midday, or night shift TI. Bob cannot work in the morning and only works when both Anna and Carlo will be working on the same day. IIL. Carlo, a trainee, can only work if his trainer Edgar will work on the same day as he. IV. Fe only takes moming shifts. V. Diego is nocturnal and does well only when Fe or Carlo works on the same day as he, 36. If Julia needs four people to work on two consecutive shifts, which of the following best qualifies for her team? (A) Anna, Bob, Diego, and Fe (B) Anna, Diego, Edgar, and Fe (©) Anna, Carlo, Bob, and Edgar (D) Bob, Carlo, Diego, and Edgar 37. Which of the following combinations of staff CANNOT be chosen to work as a team? (A) Bob, Carlo, Diego, and Edgar (B) Anna, Bob, Edgar, and Fe (©) Diego, Edgar, and Fe (D) Anna, Diego, and Fe 38. If Fe cannot make it, Edgar is compelled to report for work. Which pair ean Edgar work well with? (A) Anna and Bob (B) Bob and Carlo (©) Anna and Diego (D) Carlo and Diego BKL PhILSAT BA 0817 GO ON TOTHER NEXT PAGE 23 Alex and Brenda Reyes are celebrating their 50 wedding anniversary. They wanted to keep the celebration simple, so they invited only their children and their respective families. Alex and Brenda have three children: Charles, Darwin, and Evita, Charles got married to Ingrid, and they had two children, Greg and Heather. Greg married Lara, and they had two children: Manny and Mandy. Heather, on the other hand, is a single mother to Jack, Joy, and Julie. Darwin decided to remain single. Evita got married to a Chinese businessman by the name of Ferdie Sy. They have a daughter named Nikita, 39. How is Julie related to Brenda? (A) Her aunt (B) Her cousin (©) Her grandmother (D) Her great grandmother 40. Who of the following family members has a sumame other than Reyes? (A) Joy (B) Lara (©) Nikita (D) Darwin 41. How is Manny related to Nikita? (A) His aunt (B)_ His first cousin (©) His sister-in-law (D) His second cousin Ina certain mobile phone factory, there were five notable employees. The following are their outputs in a day: I. Albert produced the most number of mobile phones for the day. IL. Eric produced 25 mobile phones. IIL. Bobby's output consists half of Albert's IV. Carl produced ten phones less than twice of Eric's output. V. Derek produced 45 phones, the average of Albert's and Carl's. 42. Which of the following statements is INCORRECT? (A) Bobby and Eric produced the same number of phones. (B) The total number of phones the five produced is 185, (©) Carl ranks second among the top producers. (D) Derek produced more phones than Bobby. ‘BKL PhiLSAT, BA 0817 GO ON TO/THE,NEXT PAGED 24 43. How many phones did Albert produce? (A) 80 (B) 70 © 60 (D) 50 44, Which of the following statements is true? (A) Albert produced ten more than Derek. (B) Carl produced more phones than Bobby. (C) Erie and Derek are only five phones apart. (D) None of the employees produced an equal number of phones. A shelf contains six linearly arranged books, two of which were authored by Kafka, another two by Orwell, and Tolstoy and Dostoevsky wrote each of the remaining two. The books are numbered from 1 —6 indicating their position, from left to right respectively. 1. The book by Dostoevsky was in the leftmost position. II. No two books by Orwell were next to each other. III. Kafka wrote book number 6. TV. Orwell wrote book number 4. V. None of Kafka's books was next to Tolstoy's. 45. What was the book number of Tolstoy's work? (A) 5 (B) 3 (©) 2 @) 1 46. Which of the following is a book number of one of Katka's works? (A) (B) (C) (D) 47. Which of the following is a book number of one of Orwell's works? (A) 5 (B) 3 © 2 (D) 1 BKL PhILSAT BA 0817 GO ON TOTHER NEXT PAGE 25 Ina certain quiz bowl tournament, contestants are required to answer all the questions asked. Correct answers are awarded with one, three, and five points for easy, average, and difficult questions, respectively. There are ten easy questions, seven average questions, and five difficult questions. John, Josephine, Jacob, and Josh are participants in a quiz. bowl tournament. Ties are not allowed at the end of the tournament. Difficult questions will be given should there be a tie. At the conclusion, John got 60 points, Josephine got 55 points, Jacob got 43 points, and Josh got 10 points. 48. Which of the following statements is definitely true? (A) Josh was able to answer only easy questions. (B) Josh answered only one difficult question, (C) Jacob and Josephine tied at the end of the difficult round. (D) John and Josephine answered one tiebreaker question. 49. If Jacob got 46 points, it is possible that he (A) answered all the difficult and average questions (B) answered questions from either the easy round or the difficult round (C) got three correct answers from each of the rounds (D) did not get the questions in the difficult round, but answered all easy and average questions correctly 50, Based on the rules of the game, what could be the highest possible value if all of the scores from all four players are combined? (A) 224 (B) 229 (©) 231 (D) 235 ‘BKL PhiLSAT, BA 0817 GO ON TO/THE,NEXT PAGED 26 TEST C. VERBAL REASONING Directions: This section contains reading selections that are followed by a set of items. Answer the items according to what is stated or implied in the selection. Selection 1 (1) One of the greatest mysteries in art history has been solved: British academics say they have discovered the secret behind the smile of Leonardo da Vinci's Mona Lisa by studying a recently discovered portrait by the Renaissance master, La Bella Principessa (2) By comparing the techniques employed in the two works, scientists from Sheffield Hallam. University claim to have proved that the enigmatic "now you see it, now you don't” effect of the ‘Mona Lisa smile was intentional on the part of da Vinci, They have named it "the uncatchable smile." (3) The epiphany came by studs the young illegitimate daughter of a Milanese Duke, has the same effect as the Mona Lisa: from some angles the young lady seems to be smiling, from others, the smile appears to have vanished. ng La Bella Principessa. The earlier painting, which portraits (4) La Bella Principessa's mouth "appears to change slant depending on both the viewing distance and the level of blur applied to a digital version of the portrait," the researchers wrote in 8 paper published in the journal, Vision Research, according to the Telegraph. "Through a series of psychophysics experiments, it was found that a perceived change in the slant of La Bella Principessa's mouth influences her expression of contentment.” (5) Volunteers were asked to look at the painting from a variety of angles and distances. The conclusion was that, when focusing on the eyes of the painting, viewing from a distance, or when digitally blurred, a delicate smile could be seen. When viewed close up, or focusing on the mouth, however, the smile disappears. (6) The effect, evident in both paintings, was achieved by using the sfumato (which means "soft" or "pale" in Italian) technique, which uses color and shading to create an optical illusion around the mouths. (7) "The results from the experiments support the hypothesis that there is a gaze-dependent illusory effect in the portrait of La Bella Principessa," said Alessandro Soranzo of Sheffield Hallam’s psychology department. "Although it remains a question whether the illusion was intended, given Leonardo's mastery of the technique and its subsequent use in the Mona Lisa, itis quite conceivable that the ambiguity of the effect was intentional, based on explicit artistic skill and used in line with Leonardo's maxim that portraits should reflect some ‘inner turmoil of the mind.” Excerpt from: Scientists Discover the Legendary Secret Behind the ‘Mona Lisa’ Smile by Amah-Rose Abrams Retrieved from: htps://news.artnet.com/art-worldsecret-behind-mona-lisa-smile-discovered-326770 BKL PhILSAT Bl 0817 GO ON TOTHER NEXT PAGE 2 1. Which of the following organizational patterns is used in the passage? (A). Spatial (B) Process (C) Deduetive (D) Enumeration 2. Which of the following claims from the passage needs to be verified? (A) Ibis quite conceivable that the ambiguity of the effect was intentional. (B) Volunteers were asked to look at the painting from a variety of angles. (C)_ The effect, evident in both paintings, was achieved by using the sfumato, (D) A slight change in slant on the subject's mouth influences her expression. 3. Which of the following strongly supports the scientists! claim on the "vanishing" smile of Da Vinci's subject? (A) _ Da Vinci used a sfumato technique to create an optical illusion around his subjects’ mouths. (B) The illusion reiterates Da Vinci's belief that portraits must reflect a mind's turmoil. (©) LaBella Principessa that shows similar technique offered strong bases of comparison. (D) The optical illusion that creates confusion among the viewers is intended by Da Vinci, inner 4. What is the primary purpose of the passage? (A) To counter arguments debunking Da Vinci's talent (B) To solicit support for the scientists’ discovery (C)_ To demonstrate the psychophysics experiments done (D) To explain the possible reasons behind Mona Lisa's smile Selection 2 (1) Not since young Hutter arrived at Orlok’s castle in "Nosferatu" has a journey to a dreaded house been more fearsome than the one in "The Woman in Black." Both films (and all versions of Dracula") begin with the local townspeople terrified of a residence and the legends surrounding it, In this case, a green, Victorian-era attomey named Arthur Kipps (Daniel Radcliffe) is visiting a haunted house in the north of England, which can be reached only by a single-track road on a long, narrow causeway that lies so low in a brackish sea that the waters lap its edges. (2) Arthur's mission is to search the decrepit gothic mansion for the papers of its late occupant. This woman is said to haunt the house in mourning for her dead child. Local legend has it that the ghost is responsible for the deaths of other local children, brought about in rage as her form of vengeance. No wonder, then, that the locals shun Arthur, refuse him room and board, and strongly suggest he take the next train back to town. BRE PhiL SAP BA_0817, GO ON TO THE NEXT PAGES 28 (3) But Arthur must succeed because his job depends on it. His work at a firm of solicitors has been unsatisfactory, and he needs to support his son, Joseph (Misha Handley), because his wife died four years ago in childbirth. As in the Dracula legend, the carriage driver refuses to take hi close to the mansion, but a stalwart resident named Daily (Ciaran Hinds) drives him there in new motorear. (4) The house is a masterpiece of production design, crumbling, forlorn, filled with the faded and jumbled Victorian possessions of doomed lifetimes. It has a unique feature that audiences will not fail to remark upon: its own sound-effects crew. At every frightening moment, and there are many, the soundtrack paralyzes us with blasts of cacophonous noise. You wouldn't want to be in the theater next to this movie in a multiplex. (5) "The Woman in Black" is Daniel Radcliffe's first film since "Harry Potter and the Deathly Hallows: Part 2" in 2011. With a few other diversions, the "Potter" series kept him working steadily for a decade, during which he has grown taller and sprouted a crop of sideburns, but at 22, he still looks like a schoolboy — or a little young, anyway, to be the father of a 4-year-old. Nor does he have much gravitas, The film might have had more effect if his character had possessed more screen presence, but "The Woman in Black” depends mostly on the decor, location and supporting cast, some of them playing living people and some not. (6) The movie nevertheless is effective, because director James Watkins knows it isn't a character study. His haunted house is the star, The illnesses of local children provide ominous portents. Daily's wife (Janet McTeer) balances precariously on the edge of madness. And there is, a most satisfactory scene at a railroad station that might have had Daniel Radcliffe wondering if he will ever, in his career, take a routine rail journey. Review of the film ‘The Woman in Black’ by Roger Ebert Retrieved from: htp:/iwww.rogerebert.comv/reviews/the-woman-in-black-2012 5. Which of the following is the most predictable effect that the movie review has to the audience? (A) The audience would flock to the cinemas to watch. (B) Critical movie goers would consider other films. (C)_ Fans would watch this movie because of Daniel Radcliffe. (D) There will be a division among viewers, but many will agree with the review. 6. What is the dominant tone of the author in the passage? (A). Partial (B) Objective (C) Expectant (D)_Disinterested BRDEPhILSATBA_0817, GO ON TO THE NEXT PAGES 29 7. According to the author, the house is the hallmark of the movie due to its (A). overall visual appeal (B)_ mythical location (C)_ historical importance (D)_ low-cost production value 8. Which explicit features of the film is implied in the passage? 1. Predictable events IL, Fitting character actor/s IIL, Remarkable production design IV. Heightened portrayal feeling of mystery (A) only (B) Land IV only (C) I, Mand IT only (D) LIL, and IV only 9. Ifthe movie review will be encapsulated in one to two words, which of the following exacts the author's impression of the movie? (A) Good (B) Excellent (C)_ Satisfactory (D) Exceeds expectation Selection 3 (1) The One Belt, One Road (OBOR) project is the clearest expression so far of President Xi's determination to break with Deng Xiaoping's dictum to "hide our capabilities and bide our time; never try to take the lead", The Belt and Road Forum (with its unfortunate acronym, BARF) is the second set-piece event this year at which President Xi will lay out China's claim to global leadership. Its ultimate aim is to make Eurasia (dominated by China) an economic and trading area to rival the transatlantic one (dominated by America). (2) Behind this broad strategic imperative lie a plethora of secondary motivations — and it is the number and variety of these that prompts skepticism about the coherence and practicality of the project. By investing in infrastructure, President Xi hopes to find a more profitable home for China's vast foreign-exchange reserves, most of which are in low-interest-bearing American government securities. He also hopes to create new markets for Chinese companies, such as high- speed rail firms, and to export some of his country's vast excess capacity in cement, steel and other metals. By investing in volatile countries in central Asia, he reckons he can create a more stable neighborhood for China's own restive western provinces of Xinjiang and Tibet. And by encouraging more Chinese projects around the South China Sea. The initiative could bolster China's claims in that area (the “road” in "belt and road" refers to sea lanes). BRE PhiL SAP BA_0817, GO ON TO THE NEXT PAGES 30 (3) The trouble is that some of these ambitions contradict others: is a dodgy project in central Asia a better place to invest than American government securities? And with different motivations go conflicting interests. There is infighting between the most important Chinese institutions involved, including the ministry of commerce, the foreign ministry, the planning commission and China's provinces. To make matters worse, China is finding it hard to identify profitable projects in many belt-and-road countries (Chinese businessmen in central Asia call it "One Road, One Trap"). To cap it all, China is facing a backlash against some of its plans, with elected governments in Sri Lanka and Myanmar repudiating or seeking to renegotiate projects approved by their authoritarian predecessors. (4) Asa result, the forum — on the face of it a celebration of the initiative — will in reality find President Xi seeking to contain a backlash against it. That may seem to justify Europeans in their decision to stay away. But the suspicion that the project will fail could be misguided. President Xi needs the initiative because he has invested so much in problems. And Asia needs it because of an unshakeable thirst for infrastructure. The belt and road initiative has plenty of problems but President Xi is determined to push ahead with it. Excerpt from: The Economist explains by P., May 15, 2017 Retrieved from: htp://www:economist.cam/blogs/economist-explains/2017/05/economist-explains-11 10. Paragraph 2 suggests that (A)_ Doubts on OBOR arise from China's personal agenda, which center on exerting its hegemony in EuroAsia. (B) OBOR's impractical, questionable, and conflicting approach and its unclear direction discourages most European countries from joining, (C)_ China's immense investment in the infrastructure of trade routes means definite risks of unclear returns, lack of transparency, and fund deployment. (D)_ OBOR is created to unconventionally answer China's own woes and economic problems while providing infrastructure to smaller countries. 11. Which of the following will help the readers understand OBOR's internal and regional challenges stated in paragraph 2? (A) OBOR's policies, goals, bilateral/unilateral agreements (B) Sri Lanka's and Myanmar’s re-negotation policies and conditions (C) OBOR's member states, bilateral agreements, Chinese internal politics (D) Conflicting interests of China and its member countries and OBOR Forum 12, The passage is mainly concemed on OBOR's (A) benefits and disadvantages (B) potential of succeeding (C)_ threats and opportunities (D)_non-support from other countries BRDEPhILSATBA_0817, GO ON TO THE NEXT PAGES 13. The author would be LEAST likely to agree with which of the following statements on OBOR's secondary motivation? (A). China's lack of transparency and clear answers on issues of corruption, environment, and sustainability led European nations to dismiss it (B) Beyond vested interests, President Xi of China aims to build an era of harmony and trade and emphasizes cooperation and not protectionism. (©) China's platform stems from its need to speed up its slow economic growth and strengthen its global presence by reviving the Silk Roads. (D) Sri Lanka denies reports against its participation in OBOR and asserts its de which India has expressed no objections during their meeting, Selection 4 (1) Today's students — K through college ~ represent the first generations to grow up with this new technology. They have spent their entire lives surrounded by and using computers, videogames, digital music players, video cameras, cell phones, and all the other toys and tools of than 5,000 hours of their li ding, but over 10,000 hours playing video games (not to mention 20,000 hours watching TV). Computer games, email, the Internet, cell phones and instant messaging are integral parts of their lives, the digital age. Today's average college graduates have spent I (2) It is now clear that as a result of this ubiquitous environment and the sheer volume of their interaction with it, today's students think and process information fundamentally different from their predecessors. These differences go further and deeper than most educators suspect or realize, "Different kinds of experiences lead to different brain structures," says Dr. Bruce D. Perry of Baylor College of Medicine. As we shall see in the next installment, it is very likely that our students’ brains have physically changed — and are different from ours — as a result of how they ‘grew up. But whether or not this is literally true, we can say with certainty that their thinking patterns have changed. I will get to how they have changed in a minute. (3) What should we call these "new" students of today? Some refer to them as the N-{for Net]- gen or D-[for digital]-gen. But the most useful designation I have found for them is Digital Natives. Our students today are all "native speakers" of the digital language of computers, video games, and the Internet. (4) So what does that make the rest of us? Those of us who were not born into the digital world but have, at some later point in our lives, become fascinated by and adopted many or most aspects of the new technology are, and always will be compared to them, Digital Immigrants. (5) The importance of the distinction is this: As Digital Immigrants lea — like all immigrants, some better than others — to adapt to their environment, they always retain, to some degree, their "accent," that is, their foot in the past. The "digital immigrant accent" can be seen in such things BRE PhiL SAP BA_0817, GO ON TO THE NEXT PAGES 32 as turning to the Internet for information second rather than first, or in reading the manual for a program rather than assuming that the program itself will teach us to use it. Today's older folk were "socialized" differently from their kids, and are now in the process of leaming a new language. And a language learned later in life, scientists tell us, goes into a different part of the brain. (6) There are hundreds of examples of the digital immigrant accent. They include printing out your email (or having your secretary print it out for you — an even "thicker" accent); needing to print out a document written on the computer in order to edit it rather than just editing on the screen; and bringing people physically into your office to see an interesting web site rather than just sending them the URL. 1 am sure you can think of one or two examples of your own without much effort. My own favorite example is the "Did you get my email?" phone call, Those of us who are Digital Immigrants can, and should, laugh at ourselves and our "accent." (7) But this is not just a joke. It is very serious, because the single biggest problem facing education today is that our Digital Immigrant instructors, who speak an outdated language (that of the pre-digital age), are struggling to teach a population that speaks an entirely new language. Excerpt from: "Digital Natives, Digital Immigrants" by Mark Prensky, 2001 Retrieved fram:huips:www.mareprensky.com 14, How did the author describe the Digital Natives in the text? (A) Smart (B) Flexible (C) Multi-tasker (D) Disorganized 15. The author labeled the Digital Natives as such because they are (A) enslaved by technology (B)_ weakened by technology (C)_ dependent on technology {D)_ well-versed with technology 16. Digital Immigrants deal with technology by (A) adopting to what it offers (B) purchasing more expensive gadgets (C)_ telling the digital natives not to use it too much (D) promoting awareness of its hazards and ill effects BRDEPhILSATBA_0817, GO ON TO THE NEXT PAGES 33 17. Which of the following best describes the relationship between the Digital Immigrants and Digital Natives? (A). They cannot meet eye to eye on certain things. (B) They speak and understand the same languages. (C)_ They receive and process information differently. (D) They share the same passion on learning new things. Selection 5 (1) "But how many of these number one high-school performers go on to change the world, run the world, or impress the world? The answer seems to be cleat: zero." (2) The above is a quotation from Eric Barker's new book, Barking Up the Wrong Tree, where he cites the Boston University research (3) Barker's point is that while top students generally go on to be successful, few of them go on to achieve the kind of wild success most of us dream of. Instead, kids who struggle with, or don't particularly enjoy, formal education are more likely to get there. In fact, a study of 700 American millionaires found that their average GPA was just 2.9. (4) There are two potential reasons for this phenomenon, Barker writes: "Schools reward students who consistently do what they are told" — and life rewards people who shake things up. Karen Amold, the Boston University researcher, told Barker: "Essentially, we are rewarding conformity and the willingness to go along with the system." In other words, the valedictorians found out exactly what the teachers wanted and delivered it consistently. But if you think about the world’s most influential thinkers and leaders, most came up with an out-of-the-box solution to some political or scientific issue. Going along with what was already working moderately well never made anyone famous. When he visited the Business Insider office in May, Barker explained: "In school, rules are very clear. In life, rules are not so clear, So a certain amount of not playing by the rules is advantageous once you get out of a closed system like education." (5) "Schools reward being a generalist" and the real world rewards passion and expertise. Barker explains that, even if you are fascinated by history in high school, you cannot spend all your time studying the European Renaissance. At some point, you have got to stop and move onto your math homework. BRE PhiL SAP BA_0817, GO ON TO THE NEXT PAGES 34 But once you are in the working world, you will need to excel in a particular domain — and other knowledge or skills will not matter so much, And here's the real shocker: Amold found that intellectual students who genuinely enjoy learning tend to struggle in high school, They find the education system "stifling" because it does not allow them to pursue their passions deeply. (6) Barker summed up all the research nicely in the interview with Business Insider: "Valedictorians often go on to be the people who support the system — they become a part of the system — but they don't change the system or overthrow the system." (7) None of this is to say, of course, that if you were your high-school valedictorian, you will never achieve big-time success. You might very well. (8) But you'll have to keep in mind that playing by the rules will not get you as far as it once did. Taking risks and going against the grain — "sticking it to the man,” if you will —is harder to do, but itll get you farther. Excerpt from: "Why valedictorians rarely become rich and famous —and the average millionaire's college GPA is 2.9" by Shana Lebowite Retrieved fram: www.businessinsider.com 18. Which of the following statements is grounded on the selection? (A) Financial success is more achievable for average students. (B) Average students are better leamers than the top-performing ones. (C)_ Schools focus on grades and do not address the needs of average students. (D) Contrary to intellectual students, average students struggle with learning. 19. Erik Barker's book title uses an idiomatic expression that puts together his thoughts on school performance and financial success, "Barking up the wrong tree" means (A) making the wrong assumption (B)_ losing oneself in order to fit in (C)_ turing against the defenseless (D)_ promoting egocentric character 20. Which of the following statements has the weakest inclination to the author's purpose for writing the selection? (A) Education is not the only means to become a millionaire. (B) Top performing students must learn to be bold and take risks. (C)_ Class valedictorians' intelligence is not the be-all and end-all of success. (D) Average students! struggle may also bring them to financial suecess in the future. BRDEPhILSATBA_0817, GO ON TO THE NEXT PAGES 21. The main idea of the passage focuses on the reason why students (A) are not eaming millions (B) do better than other students {C) become ultimately successful (D) fail to tap their Selection 6 (1) The UN's human rights chief has attacked the "chilling indifference" to the deaths of thousands of refugees shown by European leaders as the crackdown continues across the continent. Zeid Ra'ad al-Hussein, the UN High Commissioner for Human Rights, said that although "heroic efforts" are underway to save lives in the Mediterranean, governments are turning their backs on those who survive the treacherous journey. (2) "Many ordinary people in Europe have welcomed and supported migrants, but political leaders increasingly demonstrate a chilling indifference to their fate," he told in a meeting of the UN human rights council in Geneva. (3) "Lam particularly disturbed by lurid public narratives which appear deliberately aimed at stirring up public fear and panic, by depicting these vulnerable people as criminal invading hordes." (4) The issue became a topic of debate during the EU referendum, when Nigel Farage unveiled a poster depicting migrants being escorted through Slovenia by police with the caption "breaking point". Excerpt from: UN human rights chief attacks Europe's ‘chilling indifference’ to refugees as 2017 sees record deaths by Lizzie Dearden Retrieved from: htp.//www.independent.co.uk/worldieuropetrefugee-crisis-migrants-asylum-seekers-latest-un-zeid- ‘ussein-human-rights-chilling-indifference-a7619301.htmi%e3Famp 22. Based on the passage, which of the following strongly supports al-Hussein’s claim on the chilling indifference of some European leaders? (A). Sending refugees to a campsite outside Europe (B) Refusing to take in migrants if others would not (C)_ Inciting the public's fear, panic and racial hatred (D) Showing an anti-migrant and highly racial poster 23. In order to validate the circulated claims on the crisis as an ‘invasion’ and not a refugee migration, which of the following will you check first? (A). The source and authenticity of the information (B) Unusual activities happening in the host countries, (C)_ The profile, background, and behavior of migrants (D) Unusual activities in the migrants’ country of origin BRE PhiL SAP BA_0817, GO ON TO THE NEXT PAGES 36 24. What is the main concern of the writer in the passage? (A). Presenting one side of the issue on European leaders’ indifference to the crisis (B) Condemning the leaders’ deliberate actions in order to stir public fear and panic (C) Demonstrating the depiction of vulnerable refugees as criminal invaders baseless (D)_Negating animosity due to security threats caused by cultural differences Selection 7 (1) Americans are "under siege" from disinformation designed to confuse the public about the threat of climate change, NASA's former chief scientist has said. Speaking to the Guardian, Ellen Stofan, who left the US space agency in December, said that a constant barrage of half-truths had. left many Americans oblivious to the potentially dire consequences of continued carbon emissions, despite the science being unequivocal. (2) "We are under siege by fake information that’s being put forward by people who have a profit motive," she said, citing oil and coal companies as culprits. "Fake news is so harmful because once people take on a concept it's very hard to dislodge it. (3) During the past six months, the US science community has woken up to this threat, according to Stofan, and responded by ratcheting up efforts to communicate with the public at the grassroots level as well as in the mainstream press. (4) "The harder partis this active disinformation campaign," she said before her appearance at Cheltenham Science Festival this week. "I'm always wondering if these people honestly believe the nonsense they put forward. When they say "It could be volcanoes" or "the climate always changes"... to obfuscate and to confuse people, it frankly makes me angry." (5) Stofan added that while "fake news" is frequently characterized as a problem in the right- leaning media, she saw evidence of an "erosion of people's ability to scrutinize information" across the political spectrum. "All of us have a responsibility,” she said. "There is this attitude of 'I read it on the internet therefore it must be true". (© Throughout her career, Stofan has highlighted the role of planetary science in understanding the Earth's environment and said it provided some of the most inarguable proof that atmospheric carbon dioxide leads to a warmer climate. She draws parallels between carbon emissions on Earth and the runaway greenhouse effect on Venus, a planet which once had oceans but is now a toxic infemo with surface temperatures approaching 500°C. Excerpt from: "Americans ‘under siege’ from climate disinformation former NASA chief scientist” by Hannah Devlin Retrieved from: htps:/www.theguardian,com/science/2017jun/08/americans-under-siege-from-climate- disinformation-former-nasa-chief-scientist BRDEPhILSATBA_0817, GO ON TO THE NEXT PAGES 25. According to Stofan, which of the following is the chief cause of disinformation? (A). The lack of clear cyber laws and policies (B) The presence and spreading of fake news (C)_ The people who twist facts to secure profit (D) The people's inability to scrutinize information 26. Which of the following themes captures the content of the passage? (A) The greater threat of climate change is the circulation of fake news (B) America faces problem on climate change, social media as the culprit (C) Major mining firms downplays and debunks effects of climate change (D) "Climate Change is a myth," says who and should you care verifying? 27. The closing statement of the passage will be considered a logical fallacy without which of the following supporting details? (A) Planetary scientists once presumed that whatever occurred that made Venus as hot as hell would also be expected to take place in Earth, since both planets are of the same size and have formed atmospheres of steam that kept some of the heat. (B) The Earth is not destined for an extreme scenario - even if all the carbon dioxide were burned, its oceans would not boil off completely — but Venus demonstrates the dramatic changes that can unfold when the fine balance of planet's atmosphere is tipped. (C) According to Hamana, "planets like Earth that form far enough from the sun can cool within a few million years, their steamy atmospheres condensing into liquid oceans," which is not the case with Venus that is born closer to the sun. Its oceans dried up at once. (D) The runway greenhouse, which is believed to have occurred in Venus, could begin in arth with the extreme heat of the sun that we are getting now. According to a planetary scientist Goldbatt, the increase amount of carbon dioxide emitted could make Earth unlivable. 28. According to the article, since the threat of disinformation is so harmful, in effect, the US scientific community (A) filed cases against oil and coal companies to have them pay (B) partnered with government agencies to counter fake news (C)_ pushed for fake news to be banned in social media sites (D) disclosed it to the public and the mainstream media Selection 8 (1) The 1951 United Nations Convention relating to the Status of Refugees, ratified by all European states, defined a refugee as someone who "owing to a well-founded fear of being persecuted for reasons of race, religion, nationality, membership of a particular social group or political opinion, is outside the country of his nationality, and is unable to, or owing to such fear, is unwilling to avail himself of the protection of that country.” BRE PhiL SAP BA_0817, GO ON TO THE NEXT PAGES 38, (2) International law guarantees to each person fleeing persecution the right to request asylum in a safe country. The authorities reviewing people' is a refugee or an immigrant on a case-by-case basi asylum applications determine whether one (3) Asylum laws differ in each European state because the European Union considers immigration law a matter of national sovereignty. Generally, those who are found not to qualify for asylum as refugees are deported to their country of origin. The uncertainty of the process explains why many people do not even file for asylum, but continue to live in the shadows as undocumented migrants. (4) Meanwhile, those who succeed in staying in Europe generally take unskilled jobs that the local population does not desire. In this way, they tend to fill crucial labor needs for the host society. Excerpt from: "Refugees or Immigrants? The Migration Crisis in Europe in Historical Perspective” by Theodora Dragostinova Retrieved from: hitp://arigins.ost.edu/artile/refiygees-or-immigrants-migration-crisis-ewrope-historical-perspectiv 29. In the passage, the writer is mainly concerned in (A) defining existing laws on the status of political and economic migrants in Europe (B) explaining why refugees seeking asylum would rather be an undocumented migrants (C)_ presenting the legal rights of the host country to deny entry and acceptance of refugees (D)_ showing how the refugee crisis is seen as a threat to the sovereignty of European countries 30. Which of the following statements is a clear and thorough claim on the legal rights of refugees? (A). Though the intemational law provides provision for individuals to apply for asylum in a safe country, this right is without the power as national laws and regulations of host countries exist to instantly block their entries. (B) Refugee status, as stated in international laws, obliges European countries to protect refugees from any harm and restricts them from sending them to places where there are greater risks of harm and protection. (C)_ Refugee status protects the plight of the refugees, who are unaware of their rights and who present themselves to greater danger once they flee from their countries by not honoring the asylum laws governing European states. (D) The international law cannot force European states to open their borders to refugees in times of war as the rights of the refugees are in conflict with the rights of individual states to protect their national interests from danger. BRDEPhILSATBA_0817, GO ON TO THE NEXT PAGES 31. Which of the following sentences aptly restates Paragraph | in a conversational language without changing its meaning? (A). The United Nations Convention on the Status of Refuugees, consented by all European countries, defined a refugee as someone out of fear is outside the country of his nationality, and is unable to or hesitant to seek protection due to political, racial, and economic reasons. (B) As defined by the 1951 United Nations Convention on the Status of Refugees and accepted by the European countries, a refugee is any person, who out of fear and who is unable to be protected in his country of nationality, seeks refuge in another country for political reasons. (C)_ The United Nations Convention on the Status of Refugees, which is recognized by all European countries, defines a refugee as anyone, who, out of fear, needs and prefers protection away from his country of nationality because of religious, political, and racial reasons. (D) A refuugee is defined by the United Nations Conventions on the Status of Refugees in 1951 and ratified by European nations as any person who seeks protection in another country for fear of prosecution and for the lack of protection in his own country because of political reasons. setion 9 (1) Nakamura Yoshisada, the senior research manager in Fujifilm's Pharmaceutical & Healthcare Research Laboratories, has led the development of Astalift. He was involved in film- related development for most of his career at Fujifilm, leaving him at a loss initially when he was assigned to work on cosmeties. In the end, though, he was able to draw on his previous research for the new job. (2) "People tend to think that photographic film faithfully reproduces what the eye sees, but in fact ‘memory color’ is more vivid and beautiful, thus that is what we seek in photographs," ‘Nakamura explains. "Many consumers are particularly sensitive about skin brightness and translucency in photographs. We control the coloring in photographic film so that the reproduction is a bit brighter than the actual skin color. We spent many years pursuing beautiful skin in our film imaging business, and our accumulated technologies and perspective came in handy in our cosmetics efforts, too.” (3) Photographic film and digital cameras capture light and express it as color. The company found a way to apply its cutting-edge light-analysis and control technology to its cosmetics work, too. Skin looks quite different under artificial lighting than it does in natural outdoor light, presenting cosmetics researchers with a tall challenge when it comes to producing a consistent look on the face. Fujifilm focused on optical analysis of skin condition and control of light absorption and reflection in its creation of a foundation to make the skin look beautiful under any conditions. BRE PhiL SAP BA_0817, GO ON TO THE NEXT PAGES 40 (4) For concealers, which are used to cover spots and dull areas, the traditional approach has been to layer colors on the skin to achieve the desired results. Fujifilm looked instead to light- spectrum analysis to control the absorption and reflection of the yellow light that is peculiar to skin blemishes. The company also strove to create "linking colors” that make the differences in color tone between spots and the surrounding skin less noticeable. By approaching the concealer’s task from this different angle, Fujifilm managed to come up with a product line requiring a far smaller range of color variations to match skin color. The simplicity of the lineup may be another factor contributing to its popularity among users. (5) Optical analysis of "unclear" skin that lacks translucency pointed to tiny disturbances in hese disturbances prevent found to reflect more light back and to also be uniform, so that the skin literally shines. The company ‘is to launch its Lunamer series of produets to treat the skin's epidermal layer in the epidermal layer, so small that they cannot be seen, as the culpri light from reflecting from the skin brightly and uniformly. Transparent skin, in contrast, w: July 2012. Excerpt from: Fujifilm Finds New Life in Cosmetics (Originally written in Japanese) by Ushijima Bifue Retrieved from: hitp:/‘www.nippon.com/enifeatures/e00511/ 32. Which of the following phrases strongly suggests the content of the passage? (A) From reel to real (B) History of Astalift (C)__ The science of cosmetics (D) Make-up for the photo enthusiasts 33. Which of the following statements is true according to Nakamura's claim that "many consumers are particularly sensitive about skin brightness and translucency in photographs."? (A) Brightness and translucency make pictures beautiful {B) People are upset when they look dark and dull in photos. (C)_ Some have an obsession with achieving fair complexion (D) Many people feel beautiful when they look radiant in photos. 34. Considering the language of the passage, what kind of writing would this passage likely appear in? (A) Journal (B) Magazine (C)_ Text book (D) Encyclopedia BRDEPhILSATBA_0817, GO ON TO THE NEXT PAGES 41 35. Which is the LEAST plausible reason behind Fujifilm's skin care venture? (A) Skin care business is more profitable than photography. (B) The science of photography is applicable to skin care business. (C) Skin care has become an equally demanding art as photography. (D) _ Film photography has waned at the height of digital photography. 36. Which mode of writing is dominantly utilized in the given passage? (A) Narrative (B) Persuasive (C) Expository (D) Descriptive Selection 10 (1) President Donald Trump aided Moscow's disinformation campaign during the 2016 US election by spreading false information originating from Russian state-sponsored news outlets and internet bots, a cybersecurity expert testified before Congress on Thursday. (2) "Part of the reason why active measures have worked in this US election is because the commander-in-chief has used Russian active measures, at times, against his opponents," Clint Watts, a fellow at the Foreign Policy Research Institute, told members of the Senate intelligence committee during the pane!’ first public hearing on Russian election interference since ‘Trump's inauguration in January. (3) In some instances, Trump and his campaign team propagated fake stories they appear to have learned about directly from Russian state media. Last year, then-campaign chairman Paul Manafort accused the US media of failing to cover a terrorist attack against the North Atlantic Treaty Organization air base in Incirlik, Turkey. There was no such attack — but RT (Russian intemational television network), Sputnik and pro-Russian Twitter accounts pushed a series of stories suggesting Incirlik was under threat. (4) According to Watts, pro-Russian Twitter accounts noticed Trump's loose relationship with facts and sought to capitalize on it. They "tweet at President Trump during high volumes when they know he's online and they push conspiracy theories," Watts testified. Excerpt from: Russian Disinformation Works Because Donald Trump ‘Parrots the Same Lines,’ Cyber Expert Testifies hy Jessica Schulberg Retrieved from: http:/hvww.huffingtonpost.com/entry/donald-trump-russian-active-measures disinformation us_S8dd49cbe40819403b83200 37. How are the claims in the passage organized? (A) Enumeration (B) _Deductive (C) Cause and effect (D) Compare and contrast BRE PhiL SAP BA_0817, GO ON TO THE NEXT PAGES 42 38, What logical conclusion can be drawn from the passage? (A) _ President Trump used a Russian disinformation campaign to discredit his distractors. (B) President Trump is accused of collaborating with Russia for them to interfere in the US election. (C)__ President Trump could be easily manipulated by pro-Russian accounts to buy their conspiracy theories. (D) The accusations are destabilization efforts of the other political parties to unseat President Trump. 39. Based on the selection, which logical generalization on disinformation can be draw! (A) Some influential people would resort to disinformation and deceit, controlling public opinion, to promote their own interests and agenda. (B) Through disinformation, some people can have a monopoly of truth, which would in the end make the public confused and distrusting. (C)__ Disinformation is so harmful that through a web of lies some people create, no one can tell which is true and which is not; lies are now the new truth (D) It is alarming that those who are in power are in a constant battle of creating and circulating half-truths as true, while the public is clueless. 40. Why is the claim on Trump's loose relationship with facts a logical fallacy? (A) It is not a logical argument that can be supported with reason but an attack to the person. (B) The phrasing of the claim shows prejudice; hence, it is not objective. (C)__It aims to divert the attention of the people from the real important issue, Russia's interference. (D) The speaker has the burden of proof to show Trump's inability to sort fact it would not make it true. not proving Selection 11 (1) know exactly what you mean, People misconwhattionize me all the time. (2) Man, you accidentally knock down some pig's house with a sneeze and they start telling stories about you. And now there's this little girl and her red hood, Who knows what they'll say about this one. (3) Thave self-a-team issues too. (4) Everyone is always going around saying "what a big nose you have" and "what big teeth you have." It hurts. (5) I just want to go away some place where I won't bother anyone. BRDEPhILSATBA_0817, GO ON TO THE NEXT PAGES 43 (6) They're always promising happy endings but where's my happy ending? All that happily ever after seems to be reserved for princesses and cute little animals. Especially bunnies. Why are rabbits always getting happy endings? (7) They're rodents, | tell you. Rodents! ‘The Big Bad Wolf Monologue by D. M. Larson Retrieved from: hup:/ifreedrama nethighadwolf html 41. The context of the passage suggest that the word "misconwhattionize" has something to do with (A) learning the curve (B) behaving improperly (C) thinking outside the box (D)_ having a lapse in judgment 42. Which best captures the main idea of the passage? (A) Writers should produce better stories for kids. (B) Many children stories are not at all child-friendly. (C) Parents should be concerned about what their kids read. (D) Fiction mirrors harsh realities that need to be addressed, 43, Telling his side of the story, the persona is an effective speaker because. (A) his words appeal to the emotions (B) he provides familiar backstori (C)_ there is no hint of lie in his words (D) he deserves to redeem his image 44. Which action best addresses the persona's dilemma? (A). Have rabbits as villains. {B)_ Re-tell stories from the wolf's perspective. (C)_ Find evidence that wolves are harmless animals. (D) Disregard fairy tales as they promote violence against wolves. 45. Among the following lines, which validates the persona's argument? (A). "Lhave self-a-team issues too.” (B) "They're rodents, I tell you. Rodents!" (C)_ "People misconwhattionize me all the time." (D)_ “Ljust want to go away some place where I won't bother anyone." BRE PhiL SAP BA_0817, GO ON TO THE NEXT PAGES 44 Selection 12 (1) Part of my pride in working for her was earning money I could squander: on movies, candy, paddleballs, jacks, ice cream cones. But a larger part of my pride was based on the fact that I gave half my wages to my mother, which meant that some of my earnings were used for real things— an insurance policy payment or what was owed to the milkman or the iceman. The pleasure of being necessary to my parents was profound. I was not like the children in folktales: burdensome mouths to feed, nuisances to be corrected, problems so severe that they were abandoned to the forest. I had a status that doing routine chores in my house did not provide—and it earned me a slow smile, an approving nod from an adult, Confirmations that I was adult like, not childlike (2) In those days, the forties, children were not just loved or liked; they were needed. They could earn money; they could care for children younger than themselves; they could work on the re of the herd, run errands, and much more. I suspect that children aren't needed in that way now. They are loved, doted on, protected, and helped. Fine, and yet... (3) Little by little, I got better at cleaning her house—good enough to be given more to do, much more. I was ordered to carry bookcases upstairs and, once, to move a piano from one side of 2 room to the other. I fell carrying the bookcases. And after pushing the piano my arms and legs hurt so badly. I wanted to refuse, or at least to complain, but I was afraid she would fire me, and I would lose the freedom the dollar gave me, as well as the standing I had at home—although both were slowly being eroded. She began to offer me her clothes, for a price. Impressed by these wor things, which looked simply gorgeous to a little girl who had only two dresses to wear to school, T bought a few. Until my mother asked me if I really wanted to work for castoffs. So I learned to say "No, thank you" to a faded sweater offered for a quarter of a week's pay. (4) Still, I had trouble summoning the courage to discuss or object to the increasing demands she made, And I knew that if I told my mother how unhappy I was she would tell me to quit. Then one day, alone in the kitchen with my father, I let drop a few whines about the job. I gave him details, examples of what troubled me, yet although he listened intently, I saw no sympathy in his eyes. No "Oh, you poor little thing." Perhaps he understood that what I wanted was a solution to the job, not an escape from it. In any case, he put down his cup of coffee and said, "Listen. You don't live there. You live here. With your people. Go to work. Get your money. And come on home." Excerpt from: "The Work You Do, The Person You" are by Toni Morrison Retrieved from: hup:/www.newyorker.com/magazine/2017/06/0S/the-work-you-do-the-person-you-are 46. The author is afraid to tell her mother about her perils at work because (A)_ her mother will not believe her anyway (B) she is afraid of disappointing her mother (C) there is really no intent on her part to quit (D)_ only her father sympathizes with her struggles BRDEPhILSATBA_0817, GO ON TO THE NEXT PAGES 45 47. Which of the following statements is the most INACCURATE interpretation of the father’s remark in the end? (A) ®) © (@) The value of work is less compared to the value of family. People feel the burn out when work has become life itself. Work damages one’s personal and family life. One has to learn the work-life balance. 48. Which socio-economic issue is echoed in the passage? (A) (B) © @) Housing Child labor ‘Unemployment Social services 49. Which of the following is a reason why the author refuse the faded sweater offered as payment for a week's pay? (A) (B) © @) More than anything, she felt unworthy of being given nice clothes. She would rather have new outfits, after all, she is an efficient worker. Her mother made her realize that castoffs do not compensate for her work. It gave her a sense of insecurity and self-pity knowing how lowly her work is. 50. Listed below are the author's motivation for carrying on with the house cleaning job. Which of the following arrangements shows her motivation from greatest to least? L IL. IHL. IV. (A) @®) © () Helping her family financially Being able to buy what she wants Having a sense of work efficiency ‘Taking responsibility of her decision to work 1,11, III, and IV IIL, I, IV, and Il 1,111, I, and IV U1, 1V, IIL, and 1 BRE PhiL SAP BA_0817, GO ON TO THE NEXT PAGES TEST D. 46 . QUANTITATIVE REASONING Section 1. Pattern Recognition Directions: In this test, the stem consists of a series of numbers, In each series, discover the logical rule. Select from the five choices the number that should come next in the series. 110 (A) (B) © (A) ®) © (A) (B) © 4.3 (A) ®) © 5. 100 (a) (B) © 6 845 (A) ®) © Zo (A) (B) © 25 26 29 Al 56 55 54 15 40 90 200 80 26 24 2 828 807 820 824 4 253 256 360 1 16 @) 4 ® 2 19 24 (D) 27 (E) 30 650 ) 53 (E) 52 10 5045 62 841 16 BKtuphil SATABA_ 0817 (D) 220 (E) 225 632 (D) 20 ®) 18 824-837 (D) 833 (BE) 854 19361 (D) 364 (E) 365 GO ONITO THEAEXT PAGE=C> 47 10. i 12. 13. 14. 15. 47 (A) ) © 100 (A) (B) (©) 39 (A) B) © 8 (A) (B) (©) 141 (A) @) © 14 (A) (B) © 523 (A) @) © 4 (A) (B) © 59 163 167 212 49 49 39 36 34 266 265 263 10 20 25 26 125 99 100 101 18 20 22 24 530 563 564 565 B 89 88. 87 80 94 @) «) 8136 136 133 1s 19 (D) (©) 113 105 (D) (e) © (©) 538545 (D) ®) 80 84 (@) (©) BkigPhil $AT.BA 0817 135 151 222 273 6425 26 16 266 261 255 23029 32 33 102 103 18 26 28 552559 566 567 86 92 90 Go ONTO TRENEXT PAGE==> 48 Section 2. Data Sufficiency Directions: The data sufficiency problem consists of a question and two statements, labeled (1) and (11), in which certain data are given. Decide whether the data given in the statements are sufficient for answering the question. Using the data given in the statements, plus knowledge of mathematics and everyday facts (such as the number of days in July or the meaning of the word counterclockwise), determine whether: Statement (1) ALONE is sufficient, but statement (II) alone is not sufficient to answer the question asked Statement (II) ALONE is sufficient, but statement (1) alone is not sufficient to answer the question asked. BOTH statements (1) and (11) TOGETHER are sufficient to answer the question asked, but NEITHER statement ALONE is sufficient to answer the question asked. EACH statement ALONE is sufficient to answer the question asked. Statements (1) and (II) TOGETHER are NOT sufficient to answer the question asked, and additional data specific to the problem are needed. 16. A H L IL. businessman has bought watches paid in both cash and credit card. low much will he pay in cash? The ratio of paying cash to credit card is 2: 3 The total amount paid is P45,000. (A) Statement (1) ALONE is sufficient, but statement (I1) alone is not sufficient. (B) Statement (II) ALONE is sufficient, but statement (1) alone is not sufficient, (C) BOTH statements TOGETHER are sufficient, but NEITHER statement ALONE is sufficient. (D)_ EACH statement ALONE is sufficient, (E) Statements (1) and (II) TOGETHER are NOT sufficient. A vendor charges P211.25 for § chocolate bars and 15 candies. How much does each andy cost? ‘The vendor charges P15.25 more for a chocolate bar than a candy. Each chocolate bar costs P22.00, (A) Statement (1) ALONE is sufficient, but statement (II) alone is not sufficient, (B) Statement (II) ALONE is sufficient, but statement (1) alone is not sufficient, (C) BOTH statements TOGETHER ate sufficient, but NEITHER statement ALONE is sufficient. (D) EACH statement ALONE is sufficient. (E) Statements (1) and (II) TOGETHER are NOT sufficient. BKigphil SATABA_ 0817 (GO ONTO THEANEXT PAGE=> 49 18. 20. Beginning in 2010, a movie house increased its ticket price by a constant amount each year until 2015. How much did a ticket cost in 2010? L iT (A) B) © () ) A ticket cost P175 in 2015. A ticket cost P135 in 2011. Statement (I) ALONE is sufficient, but statement (II) alone is not sufficient. Statement (II) ALONE is sufficient, but statement (1) alone is not sufficient. BOTH statements TOGETHER are sufficient, but NEITHER statement ALONE is sufficient. EACH statement ALONE is sufficient. ‘Statements (I) and (II) TOGETHER are NOT sufficient. The college accepted enrollees for BS Biology and BS Psychology this school year with a ratio of 3 : 2. How many students enrolled in BS Biology? L IL. A) (B) © @) () The total number of enrollees is 200. Forty percent of the enrollees majored BS Psychology. Statement (1) ALONE is sufficient, but statement (11) alone is not sufficient. Statement (II) ALONE is sufficient, but statement (1) alone is not sufficient. BOTH statements TOGETHER are sufficient, but NEITHER statement ALONE is sufficient. EACH statement ALONE is sufficient. Statements (1) and (II) TOGETHER are NOT sufficient. John and Doe opened a tutorial center for grade school to high school students. How many enrollees will there be on the 30" day of continuous operation? L IL, (A) (B) © () ©) ‘Two students enrolled on the first day of operation. ‘The number of enrollees increases by two every day. Statement (I) ALONE is sufficient, but statement (II) alone is not sufficient. Statement (IT) ALONE is sufficient, but statement (1) alone is not sufficient. BOTH statements TOGETHER are sufficient, but NEITHER statement ALONE is sufficient. EACH statement ALONE is sufficient. Statements (1) and (II) TOGETHER are NOT sufficient. BkigPhil $AT.BA 0817 Co ONTO THENEXT PAGE==> 21. 22. BKtuphil SATABA_ 0817 50 Marian and her friends sold cookies for the school fair. They were able to sell 352 boxes after the third week. How many boxes did they sell on the third week? ‘They sold four more boxes on the second week than on the first we IL. On the third week, the sale was twice of the second week. (A) Statement (1) ALONE is sufficient, but statement (11) alone is not sufficient. (B)_ Statement (LI) ALONE is sufficient, but statement (1) alone is not sufficient. (C) BOTH statements TOGETHER are sufficient, but NEITHER statement ALONE is sufficient (D) EACH statement ALONE is sufficient. (E) Statements (1) and (I) TOGETHER are NOT sufficient. A walk path is constructed around a circular flowerbed. What is the diameter of the flowerbed? I, The area of the circular flowerbed is 25m square meters. I, The circumference of the flowerbed is 107 meters. (A) Statement (1) ALONE is sufficient, but statement (11) alone is not sufficient. (B) Statement (11) ALONE is sufficient, but statement (1) alone is not sufficient. (C) BOTH statements TOGETHER are sufficient, but NEITHER statement ALONE is sufficient. (D) EACH statement ALONE is sufficient. (E) Statements (1) and (I) TOGETHER are NOT sufficient. A mother gives her son a cash reward for every passing final grade that he gets at the end of the semester. How much will her son receive at the end of the semester? 1. _ Each passing grade has a corresponding cash reward as shown below. Grade’ 1.00- 1.25 [1.50— 1.75 [2.0-2.25 [250-275] 3.00 Reward (in Peso) [__1,000 500 300, 200 100) Il, The Grade Point Average (GPA) of her son is 1.68. (A) Statement (1) ALONE is sufficient, but statement (11) alone is not sufficient. (B)_ Statement (LI) ALONE is sufficient, but statement (1) alone is not sufficient. (C) BOTH statements TOGETHER are sufficient, but NEITHER statement ALONE is sufficient (D) EACH statement ALONE is sufficient. (E) Statements (1) and (I) TOGETHER are NOT sufficient. GO ONITO THEAEXT PAGE=C> 51 24, Aubrey and Denise worked as consultants at P700 and P800 per hour, respectively. How many hours did each of them work for a month? Aubrey worked 10 hours more than Denise in a month, I, Their total income for a month is P157,000. (A). Statement (J) ALONE is sufficient, but statement (11) alone is not sufficient, (B) Statement (L1) ALONE is sufficient, but statement (1) alone is not sufficient. (C) BOTH statements TOGETHER are sufficient, but NEITHER statement ALONE is sufficient (D) EACH statement ALONE is sufficient. (E) Statements (1) and (LI) TOGETHER are N OT sufl nt, 25. A psychologist conducted a free seminar held at a university auditorium. For documentation purposes, the school administrator determines the number of attendees. How many attended the seminar expressed at a percent of capacity? Thirty percent of the auditorium is not occupied. II The auditorium has 450 seats, of which 315 are occupied. (A) Statement (1) ALONE is sufficient, but statement (II) alone is not sufficient. (B) Statement (II) ALONE is sufficient, but statement (1) alone is not sufficient. (C)_ BOTH statements TOGETHER are sufficient, but NEITHER statement ALONE is sufficient. (D) EACH statement ALONE is sufficient. (E) Statements (1) and (II) TOGETHER are NOT sufficient. 26. Coach Benedict would like to determine the average height of his swimmers. What is the average height of his swimmers in centimeters? I. He trains 5 swimmers of different heights. I, The measured heights of his swimmers are 5'3", 5'7", 5'5", 5'10", and 5'11". (A) Statement (1) ALONE is sufficient, but statement (II) alone is not sufficient. (B) Statement (I) ALONE is sufficient, but statement (1) alone is not sufficient, (C) BOTH statements TOGETHER are sufficient, but NEITHER statement ALONE is sufficient. (D) EACH statement ALONE is sufficient. (E) Statements (1) and (II) TOGETHER are NOT sufficient. BkigPhil $AT.BA 0817 Co ONTO THENEXT PAGE==> 27. 28, 29, 52 A jeweler designed a pair of triangle-shaped dangling earrings. What is the total perimeter of the triangular earrings? Each of the two sides has a length of 2 cm, and the length of the third side is 1 cm. Il, The sum of length of all sides of an earring is $ cm. (A) Statement (1) ALONE is sufficient, but statement (II) alone is not sufficient. (B) Statement (II) ALONE is sufficient, but statement (1) alone is not sufficient, (C) BOTH statements TOGETHER are sufficient, but NEITHER statement ALONE is sufficient. (D) EACH statement ALONE is sufficient. (E) Statements (1) and (II) TOGETHER are NOT sufficient. A travel agent booked a family of five (3 adults and 2 kids) to a nature trip in Sagada for three days and two nights. He arranged the transportation, accommodation and three meals per day for the family. If the agency charges 10% of the total fee for the package tour, how much will the agency receive? 1, The total cabin accommodation and meal expense per adult is P2,500, and per child is P1,500. Il, The transportation cost from Manila to Sagada and back costs P720 per head. (A) Statement (1) ALONE is sufficient, but statement (II) alone is not sufficient. (B) Statement (II) ALONE is sufficient, but statement (1) alone is not sufficient. (C)_ BOTH statements TOGETHER are sufficient, but NEITHER statement ALONE is sufficient. (D)_ EACH statement ALONE is sufficient. (E) Statements (1) and (II) TOGETHER are NOT sufficient. Roel and Rod are playing marbles. Each one tries to shoot his marble on a hole on the ground, Unfortunately, none of them was able to shoot a marble on their first try. How far is the distance of Roel's marble to Rod's? 1. Hypothetically, the location of Roel’s marble is (3,4) while Ro both measured in feet. I Roel's marble is 2 feet away from the hole on the ground, marble is (5,2), (A) Statement (1) ALONE is sufficient, but statement (II) alone is not sufficient. (B) Statement (11) ALONE is sufficient, but statement (1) alone is not sufficient. (C) BOTH statements TOGETHER are sufficient, but NEITHER statement ALONE is sufficient. (D)_ BACH statement ALONE is sufficient, (E) Statements (1) and (II) TOGETHER are NOT sufficient. BKigphil SATABA_ 0817 (GO ONTO THEANEXT PAGE=> 53 30. The backyard of Mr. Jan's farmhouse is to be fenced with wood planks only on three sides. The wood planks are 10 feet in length and are nailed horizontally to wooden posts. ‘What is the total perimeter of the backyard? |. The backyard of the farmhouse is rectangular in shape. II One hundred fifty wood planks cover the entire perimeter of the three sides of the backyard, with 100 wood planks on one side and 25 wood planks on each parallel sides. (A) Statement (1) ALONE is sufficient, but statement (11) alone is not sufficient. (B) Statement (I) ALONE is sufficient, but statement (1) alone is not sufficient. (©) BOTH statements TOGETHER are sufficient, but NEITHER statement ALONE is sufficient. (D) EACH statement ALON {E) Statements (1) and (11) TOGETHER are NOT sufficient. Section 3. Data Interpretation Directions: In this section, some sets of data are given. Interpret or analyze the given data to answer the questions that refer to the data presented. Select the correct answer from the options given, For items 31 and 32, refer to the following situation. Four competing appliance stores put a television set of the same brand on sale as shown in the table below, ‘Store Original Price (P) Discount Rate (%) TVA 11,500 8 TVB 10,000, 15 TVC 12,000, 20 TVD 9,000 5 31. Which of the following stores offers the biggest discount from its original price? (A) TVA (B) TVB (©) TVC (D) TVD 32. What is the selling price of the television set in store TVD? (A). P8,550.00 (B)_P8,600.00 (©) P8,995.00 (D) 9,000.00 BkigPhil $AT.BA 0817 Co ONTO THENEXT PAGE==> For items 33 to 36, refer to the graph below. 54 A local zoo has an interesting collection of species of exotic birds. Below is a table showing the number of birds and approximate weights according to classi 34, 36. ation, ‘Approximate Weight per Exotic Bird Species No. of Birds Bird (in kilograms) Rainbow Lorikeet 4 0.20 Lear's Macaw 6 0.90 Kingfisher 5 0.03 Northern Cardinal 5 0,04 What is the modal weight of the birds? (A) ®) © (@) ‘What is the probability of selecting a Kingfisher? 0.90 0.29 0.20 0.05 (A) 0.03 B) (©) 0.05 0.15 (D) 0.25 ‘What is the ratio of the number of Rainbow Lorikeet to the number of Lear's Macaw? (A) (B) © () sane Baek Which of the following statement(s) is(are) true based on the data above? 1 IL UL. 1, (A) (B) © () The average weight of the Northern Cardinal birds is 0.20. 1's Macaw birds is 0.90. Rainbow Lorikeet is the least preferred exotic bird in the zoo. Kingfisher has the lightest weight among the exotic birds ‘The average weight of the L Tonly IL only Tand IV only and IV only BKtuphil SATABA_ 0817 GO ONTO THEAEXT PAGE=C> 55 For items 37 and 38, refer to the graph below, DISTRIBUTION OF THE CANCER PATIENTS BY STAGE AND AGE INTERVAL IN A CERTAIN HOSPITAL (N = 25) HSS10<65 H45t0<55 M35 t0<45 mW Vv Stage of Breast Cancer 0 1 2 3 4 5 Diagnosed Patients 37. What fraction of the total number of patients diagnosed with breast cancer is in the age interval 35 to less than 45 years old? 1 a~ay 2 (A) 5 fo 25 2 25 22 25 (B) © () 38. What is the probability that the patient selected at random is diagnosed to be in stage 1 breast cancer? (A) 0.48 (B) 0.25 (C) 0.20 (D) 0.12 BkigPhil $AT.BA 0817 Co ONTO THENEXT PAGE==> 56 For item 39 and 40, refer to the following situation. A research on pizza restaurant preference near the university belt was conducted. One hundred twenty five students were surveyed. The results showed that most students preferred either of the three: Red Peppers, Mr. Cheddar's, and Crusty's. Below is a diagram showing the number of students who prefer these pizza restaurants. Red Peppers Mr. Cheddar's CLL Crusty's 39. What percent of students prefer Red Peppers and Mr. Cheddar's but not Crusty's? (A) 8% (B) 12% (C) 15% (D) 30% 40. How many students prefer pizza restaurants other than Red Peppers, Mr. Cheddar's, or Crusty's? (A) 9 (B) 29 (C) 44 (D) 96 BKigphil SATABA_ 0817 (GO ONTO THEANEXT PAGE=> 37 For items 41 to 43, refer to the following situation. PERCENTAGE OF CHILDREN WHO RECEIVED DENTAL CARE, Poverty Status BY POVERTY STATUS IN 2012 (N = 2,500) Less Than 6 Months [Between 6 Months and 1 Year mBetween | and 2 Years Between 2 and 5 Years |More Than 5 Years Extremely Poor Poor Nonpoor 0% 20% 40% 60% 80% 100% Percent of Children 41. How many children were reported to be in between 2 and 5 years of age? (A) 75 (B) 60 (©) 55 (D) 40 42. Which of the following statement(s) is(are) true? I. The total number of children under the extremely poor status is equal to the number of children in the poor status. IL. For the nonpoor status, the number of children with the ages 6 months to 1 year is greater than the number of children with ages 1-2 years and 2-5 years combined. IIL The 2.5% of the children with ages 1 to 2 years belong to the poor status. (A) Lonly (B) Monly (C) Land Il only (D) Mand Il only BKluPhil SATABA_ 0817 43. 58 Which of the following statements is a CORRECT analysis of the graph? (A) (B) © (@) Based on the report, there are approximately 11.3% of the children from 2 to 17 years old, who received dental care in 2012. ‘The average number of children who have had a dental visit between 6 months and 1 year is approximately 18% Children in more than 5 years of age bracket, and are living in households with a poor status received dental care the most compared to those in other poverty statuses. Families in the extremely poor status are less likely to have had dental visit in 2012 than those families with nonpoor status For items 44 to 46, refer to the following situation. Mr. Reyes supplies rice to three grocery stores in Davao City. The table below shows the number of sacks of rice sold in each of their three branches over a period of six months. 44, 4s. 46. Jan_[ Feb [Mar | Apr | May [ Jun | Total BranchA | 63 [57 [58 [ 62 | 69 [51 | 360 Branch B | _52_| 46 | 54 [43 [ 49 | 53 | 297 BranchC | 46 [66 [ 52 | 54 | 60 | 65 [343 Total | 161 [ 169 | 164 | 159 [ 178 | 169 [1000 Which month has the biggest increase in total sales from the previous month? (A) (B) © () February March April May ‘What is the average sales for Branch A for the first four months of the year? (A) (B) © (@) 3575 58.75 60.0 62.5 Which of the following inferences is NOT true based on the given data? (A) @) © (@) Branch A sold more sacks rice than Branch B. Branch B is the least visited store among the three. Branch C contributed 34.3% to the total number of sacks sold over the period of six months Mr. Reyes received his highest sales during the month of May. ‘BKLPRILSAT BA 0817 GO ON TO' THE NEXT PAGE.> 59 For items 47 to 50, refer to the chart below. 47. 48. 49, COMPARISON OF CAUSES OF DEATH FOR MUSICIAN OF DIFFERENT GENRES (N = 600) Rap Hip hop Heart-related Suicide. 6.9% Cancer 7.6% corer 1% Heart-related Suicide rel 6.2% Adapted from hep:/theconverstion.com/music-to-die-forhow-genr-affees-popular-musicians-ife-expectancy-36660 Which of the following causes of death has a percentage difference that is twice to that of the suicide rate for the two groups of musicians? (A) Accidental (B) Homicide (©) Cancer (D) Others Which of the following is the LEAST cause of death for hip hop musicians? (A) Accidental (B) Homicide (C) Suicide (D) Cancer Supposing half of the population are rap musicians, how many of them died from homicide? (A) 165 (B) 155 (©) 133 (D) 306 BREPhLSAT_BA O81, ‘GO ONTO THE NEXT PAGE 60 50. Which of the following interpretations on the causes of death for rap and hip hop musicians is(are) true based on the data? L I, UL. (A) B) © (@) Homicide is the leading cause of death, with a di the two groups. Cancer and heart-related mortality rate are similar to both groups of musicians, Suicide, accidental, and homicide are greater in percentage for the hip hop group than for the rap group. rence of 5% in rate for Tonly Monly Ionly Tand IIT only STOP! WAIT FOR FURTHER INSTRUCTIONS. BKluPhil SATABA_ 0817 PB PhiLSAT_BA Copyright © 2017 by the Center for Educational Measurement, Inc. All Rights Reserved. No part of the content of this assessment publication may be reproduced in any form or by any means, including electronic storage, reproduction, execution or transmission.

You might also like